If the pattern below follows the rule "starting with 10, every consecutive line has a number 1 less than the previous line," how many marbles must be in the seventh line?

If The Pattern Below Follows The Rule "starting With 10, Every Consecutive Line Has A Number 1 Less Than

Answers

Answer 1

Answer: B: 4

Step-by-step explanation: just took the test

Answer 2

The seventh line will be having 4 marbles which can be determined by taking the given data as an arithmetic progression with the first term as 1 and the common difference -1. Hence, option B is the right choice.

What is an Arithmetic Progression?

An arithmetic progression (A.P.) is a series where every term is the sum of the previous term and a common difference.

The first term of an arithmetic progression is denoted by a, and its common difference is denoted by d. The n-th term of an arithmetic progression can be found using the formula

n-th term = a + (n-1)d.

How do we solve the given question?

We are given a pattern, starting with 10 marbles and every consecutive line has 1 less marble than the previous line. We are asked to find the number of marbles in the 7th line.

This can be seen as an arithmetic progression with the first term a = 10, and the common difference d = -1.

We are asked to find the number of marbles in the 7th line. This can be taken as the 7th term of the given arithmetic progression.

n-th term = a + (n-1)d

or, 7th term = 10 + (7-1)(-1) = 10 + 6(-1) = 10 - 6 = 4.

∴ The seventh line will be having 4 marbles which can be determined by taking the given data as an arithmetic progression with the first term as 1 and the common difference -1. Hence, option B is the right choice.

Learn more about an arithmetic progression at

https://brainly.com/question/6561461

#SPJ2


Related Questions

Point M is the midpoint of line AB. If M is at (6,-2) and A is at (-3,0), what are the coordinates of endpoint b?

Answers

Answer:

Step-by-step explanation:

(x + 6)/2 = -3

x + 6 = -6

x = -12

(y - 2)/2 = 0

y - 2 = 0

y = 2

(-12, 2)

What is the domain and range in this graph?

Answers

Step-by-step explanation:

The graph starts when the value of x is -1 and ends when it's value is 5 , So domain is [-1,5]

The highest value of the graph is 4 which is at x = -1 and the lowest value of the graph is 0 , which is at x = 5, So range is [0,4]

The domain is equal to all the input values (x-values)

The range is equal to all the output values (y-values)

In this problem,

The domain: [-1, 5]

The range: [0, 4]

What is the value of y in the equation 5x + 2y = 20, when x = 0.3?

Answers

Answer:

9.25

Step-by-step explanation:

5(0.3) = 1.5

20-1.5=18.5

18.5÷2=9.25

Answer:

y = 9.25

Step-by-step explanation:

5x + 2y = 20

when x = 0.3 we have

5(0.3) + 2y = 20

1.5 + 2y = 20

Using the subtraction property subtract 1.5 from both sides of the equation

That's

1.5 - 1.5 + 2y = 20 - 1.5

2y = 18.5

Divide both sides by 2

[tex] \frac{2y}{2} = \frac{18.5}{2} [/tex]

We have the final answer as

y = 9.25

Hope this helps you

Each team earns 1 point for each person who participated in the track-and-field day. Who wins the meet---the eighth graders or the seventh graders?

Answers

Answer:

THE SEVENTH GRADERS

Step-by-step explanation:

next time give more context tho, but the seventh graders got 712 points while the eighth graders got 332 points. love from granny smith y'all!

Answer:

7th

Step-by-step explanation:

sup

Solve to 3 s.f and in scientific notation (5.11×10^-4)+(4.33×10^-3)

Answers

Answer:

0.00484

Step-by-step explanation:

Write an equation in slope intercept form for the line that passes through the point with the given slope. point: (4,2) and m=3

Answers

Answer:

3x/10 - y/10 = 1

Step-by-step explanation:

(y-2)/(x-4)=3

y-2=3x-12

y-3x= -12

3x/10-y/10=1

Find R and theta with the coordinates (-3,7).

Answers

"R" is the distance from the origin to the point.

R = √(-3² + 7²)

R = √(9 + 49)

R =  √(58)

R = 7.62

tangent (Θ)  =  7 / -3

Θ  =  arctan (7 / -3)

Θ  =  113.2°  counterclockwise from the x-axis

please answer this for me

Answers

Answer:

B (5,2)

Step-by-step explanation:

First person to answer gets brainliest Find the domain and range of the following absolute value functions f(x)= |-2x|+7

Answers

Answer:

domain: all real numbers

range : y ≥ 7

Step-by-step explanation:

The domain is the values that x can take

X can take any value so the domain is all real numbers

The range is the values that y can take

The smallest value  for the absolute value is 0

0+7 =7 so the smallest value for y is 7

range is y ≥ 7

The team manager for a high school basketball team discarded a lower
outlier and calculated that the mean number of points scored per game
was 45 and the median was 48.
What is the likely effect if the analyst decides to include the lower outlier in the
calculations?

a) The median will remain about the same, but the mean will decrease.
b) The median will decrease, and the mean will increase.
c) The mean will stay about the same, but the median will decrease.
d) The median and mean will both stay about the same.

Answers

Answer:

The median will remain about the same, but the mean will decrease.

Step-by-step explanation:

Cus i said so

Answer:

The median will remain about the same, but the mean will decrease.

Edge 2020

-25 = 1/2 (10x - 2) + 3x what is x?

Answers

Answer:

x=[tex]-\frac{1+2i\sqrt{31} }{5}[/tex]

Step-by-step explanation:

Answer:

-3

Step-by-step explanation:

−25=  [tex]\frac{1}{2}[/tex]  (10x−2)+3x

−25=([tex]\frac{1}{2}[/tex])(10x)+( [tex]\frac{1}{2}[/tex] )(−2)+3x(Distribute)

−25=5x+−1+3x

−25=(5x+3x)+(−1)(Combine Like Terms)

−25=8x+−1

−25=8x−1

Step 2: Flip the equation.

8x−1=−25

Step 3: Add 1 to both sides.

8x−1+1=−25+1

8x=−24

Step 4: Divide both sides by 8.

8x  ÷ 8   =   −24  ÷ 8

Find the value of x.
A. 13
B. 16
C. 10
D. 17

Answers

Answer:

C.10

Step-by-step explanation:

Since the angels are vertical we know hey are congruent so set them equal to each other.First subract 3 from both sides. The equation now is 60=6x. Next divde both sidesz by 6. The equation now is x=10.

I hope this helped and have a great day!

(Please vote this the brainliest)

Answer:

it's C. 10

Step-by-step explanation:

6(10)+3 = 63

6(13)+3 = 81

the answer need to equal the same amount as the top one which is 63 because that's a vertical angle

7 students have 3 books each. Which helps you find the total number of books?

Answers

Answer:

21 Books in total

Step-by-step explanation:

If you just want the total number of books, here:

If each student has 3 books, and there are 7 students, we can create the equation: 7 * 3 :because this is showing that 7 people each have 3 books

7 * 3 = 21 so There are 21 books in total

Another we could write it is 3 + 3 + 3 + 3 + 3 + 3 + 3 because there are 7 students with 3 books each. Adding this up would equal 21.

Answer:

21 books in total

Step-by-step explanation:

If there is 7 students who have 3 books each, and want to find the total. The easiest way is multiply 7 with 3 to give you an answer of 21

Or draw a picture if it helps

Draw 7 stick figure and under it draw 3 books for them and count the books, still gives you 21

I think this is what you want

Solve by elimination:
4x+9y=28
-4x-y=-28

Answers

Step-by-step explanation:

4x+9y=28

-4x-y=-28

8y=0

divide both side by 8

y=0

solve for X in equation 1

4(X)-9(0)=28

4x-0=28

4x =28

divide both side by 4

4x/4=28/4

X= 7

y=0,x=7

Your car's gas tank is 3/4 full before you leave for your trip. When you stop at the gas station, the tank is only 1/12 full. Show the amount of change as a rational number.

Answers

Answer:

2/3

Step-by-step explanation:

The telephone company offers two billing plans for local calls. Plan 1 charges ​$32 per month for unlimited calls and Plan 2 charges ​$17 per month plus ​$0.03 per call.

Answers

Answer:

The indifference point is 500 calls.

Step-by-step explanation:

Giving the following information:

Plan 1 charges ​$32 per month for unlimited calls and Plan 2 charges ​$17 per month plus ​$0.03 per call.

I assume we have to determine the indifference point between the two plans.

First, we need to structure the cost formulas:

Plan 1= 32

Plan 2= 17 + 0.03*x

x= call

Now, we need to equal both formulas:

32 = 17 + 0.03x

15= 0.03x

500= x

The indifference point is 500 calls.

Plan 1= $32

Plan 2= 17 + 0.03*500= $32

Answer:

Step-by-step explanation:

SOlve plz!!! I need help!! I'm trying to get this done!! I'll DO ANything

Answers

Answer:

3

Step-by-step explanation:

How many times does -6 go into -18?

-6x3=-18

-6 goes into -18 3 times.

So:

b=3

Bam

Find the distance between the two points rounding to the nearest tenth (if necessary)

(-3,3) and (-1,5)​

Answers

Answer:

The answer is

2.8 units

Step-by-step explanation:

The distance between two points can be found by using the formula

[tex]d = \sqrt{ ({x1 - x2})^{2} + ({y1 - y2})^{2} } \\ [/tex]

where

(x1 , y1) and (x2 , y2) are the points

From the question the points are

(-3,3) and (-1,5)

The distance is

[tex]d = \sqrt{( { - 3 + 1})^{2} + ({3 - 5})^{2} } \\ = \sqrt{ ({ - 2})^{2} + ( { - 2})^{2} } \\ = \sqrt{4 + 4} \\ = \sqrt{8} \: \: \: \: \: \: \: \: \\ = 2 \sqrt{2} \: \: \: \: \: \: \\ = 2.82842[/tex]

We have the final answer as

2.8 units to the nearest tenth

Hope this helps you

A bottle of hot sauce is 3/4 full. Leigh uses 2/9 of the contents of the hot sauce bottle for lunch. How much of a full bottle of hot sauce did Leigh use for lunch? A)26/36 B)5/36 C)1/6 D) 1/2 E) 1/3. PLEASE HELP.

Answers

Answer:

  C)  1/6

Step-by-step explanation:

2/9 of 3/4 is ...

  [tex]\dfrac{2}{9}\times\dfrac{3}{4}=\dfrac{2\cdot 3}{9\cdot 4}=\dfrac{6}{36}=\boxed{\dfrac{1}{6}}\qquad\text{matches C}[/tex]

Answer:

hlp

Step-by-step explanation:

Mixed nuts sell for $7.89 a pound. Leticia fills a bag and weighs it. The bag weighs 2.3 pounds. About how much will the bag of nuts cost? Estimate $7.89(2.3). The bag of nuts will cost about .

Answers

Answer:

the answer is $16

hope this helps

please i need help with these problems thank you!!

Answers

First pic power of -2
Second pic the power of y and x is -3

Choose the correct equation to represent the number line model.

Answers

Answer:

-5 + -4 = -9

Step-by-step explanation:

-5 + -4 = -9

Start at 0.

You will need to move 5 spaces to the left (Negative numbers move to the left while positives go right).

And -4. Move 4 spaces left again. There you land at -9.

The plus sign is just to combine -5 and -4 together.

Thus the answer is:

-5 + -4 is -9.

Here’s why the other equations aren’t correct.

The first equation says “5 + 4 = 9”.

To represent this, you would move 5 spaces to the right (because the number is posititve.)

Then combine 4. Which you move 4 spaces to the right again. This leaves you at 9 instead of -9.

The second equation says “-5 + -9 = -14”.

To represent this, you would move 5 spaces to the left.

Then move 9 spaces to the left again. Which leaves you at -14.

Your final destination should be -9. Not anywhere else.

Evaluate (2 - 5 i) (p + q) (i) when p = 2 and q = 5 i

Answers

Answer:

29i

Step-by-step explanation:

https://socratic                 .org/questions/how-do-you-evaluate-2-5i-p-q-i-when-p-2-and-q-5i#304498

If you want a description, copy that into your browser and take out the spaces between "socratic" and ".org".

Answer:

A

Step-by-step explanation:

-2x+5=24 I need help!!!

Answers

Answer:

[tex]x=-\frac{19}{2}[/tex]

Step-by-step explanation:

hope that was the answer you were looking for :/

Answer:

x = -19/2

Step-by-step explanation:

[tex]-2x+5=24[/tex]

Then subtract 5 from 24

[tex]-2x=19[/tex]

Then divide -2 by 19 to get:

[tex]x = -\frac{19}{2}[/tex]

Hope this helps!

Please help brainiest

Answers

the measure is d which is 65

Answer: The answer is C. 60

Step-by-step explanation:

you have to subtract the known angles to find the missing angles opposite angle. 180-60-70=130 then look at the line KM, it has to equal 180 so subtract the angle we just got to get the other angle 180-130=60

One less than the product of four and a number.
(Algebraic expression)

Answers

Answer:

1-4x

Step-by-step explanation:

The area of a square poster 26 inch. 2. Find the length of one side of the poster to the nearest tenth of an inch.

Answers

Answer:

6.5 inches

Step-by-step explanation:

Divide 26 by 4

26/4=6.5

The length of one side of the poster to the nearest tenth of an inch is 5.1 inches.

What is the area of a square?

The area of a square can be calculated as the square of the sides of a given figure.

The area of a square = side x side

Given; The area of a square poster 26 inch

We need to find the length of one side of the poster to the nearest tenth of an inch.

The area of a square would be S^2

26 = S^2

s = sqrt(26) = 5.099

Then rounded to nearest tenth = 5.1 inches

Hence, the length of one side of the poster to the nearest tenth of an inch is 5.1 inches.

Learn more about the area here;

https://brainly.com/question/1658516

#SPJ2

Is it true that square roots can be compared and ordered by comparing and ordering the numbers underneath the radical symbol?

Answers

Answer:

Yes

Step-by-step explanation:

I had this same question on a quiz and it is correct!

The square root of a number is equal to another number that when squared is equal to the original number.

Now we want to see if Is it true that square roots can be compared and ordered by comparing and ordering the numbers underneath the radical symbol, and we will see that yes, this is true.

To see that this is true:

For example, if the A and B are two numbers larger than 0, such that:

A > B

Then:

√A > √B

For example let's use A = 9 and B = 4.

9 > 4

And obviously the roots are:

√9 > √4

3 > 2

So yes, we can compare and order square roots by comparing the numbers inside of it.

If you want to learn more, you can read:

https://brainly.com/question/1387049

Johnathon is planting potatoes and carrots in his
garden. He has 30 potato plants and 50 carrot plants.
He wants to distribute the two types of plants equally
among the rows.
What is the greatest number of rows he can make?

Answers

Answer: 3

Step-by-step explanation: well if he does rows of 10 then it will have to be 3   and he will have 20 potato plants left over.

i need helpppppppppppppppppppp

Answers

Answer:

6

Step-by-step explanation:

Other Questions
_________ attacks are becoming less common in modern operating systems. a. Denial of service b. SYN flood c. Buffer overflow d. None of the above 6. Do criminalists make truly scientific and objectivedeterminations? What claim can you make about the relationship between energy and mass? Walking is not the most exciting form of exercise a person can take on for fitness. Yet it is low impact and requires no fancy equipment. That makes walking a simple activity for those new to exercise. Walking is less likely than other exercises to cause injury to leg tendons and muscles. All it requires is a good pair of sneakersno cables, stylish outfits, or weights required. Of course, a person could increase the benefit of a workout by adding weights or speed walking.How does the third sentence ("That makes walking...") support the main idea of the paragraph? (5 points) aIt points out that walking is a good introduction to exercise for beginners. bIt says that walking is too easy for people who have experience with exercise. cIt points out that beginning exercisers may not like walking because it is too easy. dIt says that walking is less costly than other activities because it needs no equipment. 4. m stands for....a) millimeterC) centimeterb) meterd) Liter Approximately 2,094,800 people took avacation to a state park this year. 25years ago 158,975 people vacationed tostate parks. Approximately how manymore people visited the park than 25years ago? Fourteen less than the quotient of a number and 5 Ty_1st 9 wksRegion of Birth of Foreign-Born Population of the US Which statements are true about the number ofEuropean immigrants between 1900 and 1990? CheckRegion1900 1930 1990 all that apply.Europe 8,881,548 11,784,010|4,350,403 Half as many people were from Europe in 1900than in 1930.Asia 120.248 275,665 4,979,037 Twice as many people were from Europe in 1900than in 1990.Africa 2,538 18,326 363.819 The European-born population increased between1900 and 1930.LatinThe European-born population decreased between137,458 791,840 8,407,8371930 and 1990.AmericaThe European-born population decreased between1900 and 1990.Done how to write on a football match between two teams How does the arrangement of atoms differ in crystals and glass? Samantha claims 3/5 is an integer is she correct? 1. Consider this statement: The immigration from 1901 to 1910 had a greater effect on the United States than the immigration from 20012010. Why might this statement be true? 2. What caused the large drop from 19301950? 3. Approximately how many immigrants came to the United States between 1890 and 1920? What is the measure of the smallest angle in the figure? The figure is not drawn to scale. Fox is related to cunning, as leaf is related to1. Calcium2. hub3. sap4. crump 66.5 divided by 7/2I WILL MARK BRAINLIST!!! Solve the equation. -1 + x = -10 x = What is the word for when a Danter moves one part of their body while the rest of the body remains still? 5 consequences of concentrated power Evaluate x^3 y^2 if x = 2 and y= 3/4 Look at the screenshot pls!!!!!!!!!!!!!!!!!!!!!!!! THANK YOU